Last visit was: 23 Apr 2024, 12:55 It is currently 23 Apr 2024, 12:55

Close
GMAT Club Daily Prep
Thank you for using the timer - this advanced tool can estimate your performance and suggest more practice questions. We have subscribed you to Daily Prep Questions via email.

Customized
for You

we will pick new questions that match your level based on your Timer History

Track
Your Progress

every week, we’ll send you an estimated GMAT score based on your performance

Practice
Pays

we will pick new questions that match your level based on your Timer History
Not interested in getting valuable practice questions and articles delivered to your email? No problem, unsubscribe here.
Close
Request Expert Reply
Confirm Cancel
SORT BY:
Date
Tags:
Difficulty: 655-705 Levelx   Arithmeticx   Word Problemsx               
Show Tags
Hide Tags
VP
VP
Joined: 15 Dec 2016
Posts: 1376
Own Kudos [?]: 207 [0]
Given Kudos: 189
Send PM
Tutor
Joined: 04 Aug 2010
Posts: 1315
Own Kudos [?]: 3134 [1]
Given Kudos: 9
Schools:Dartmouth College
Send PM
Senior Manager
Senior Manager
Joined: 13 Mar 2021
Posts: 338
Own Kudos [?]: 101 [0]
Given Kudos: 227
Send PM
Director
Director
Joined: 16 Jun 2021
Posts: 994
Own Kudos [?]: 183 [0]
Given Kudos: 309
Send PM
Re: Joanna bought only $0.15 stamps and $0.29 stamps. How many $0.15 stamp [#permalink]
Statement 1:
15x+29y = 440

15x + 29x = 440
44x = 440
x=10, y=10

solution for 15x+29y=440 is as follows:
x=10, y=10
other solutions for the same was not available
SUFFICIENT.

Statement 2:
x=y
Here, x and y can take any values
Therefore INSUFFICIENT.

Hence IMO A
Intern
Intern
Joined: 29 Jan 2022
Posts: 11
Own Kudos [?]: 4 [0]
Given Kudos: 7
Send PM
Re: Joanna bought only $0.15 stamps and $0.29 stamps. How many $0.15 stamp [#permalink]
Magophon wrote:
Here is the method to never fail to answer correctly Diophantine-equations-related Data Sufficiency problems.

1. First, be sure that the 2 variables must be non-negative integers or positive integers and that each statement provides a linear equation relating the 2 variables. Furthermore, be sure that the 2 equations are not equivalent (2x+3y=20 and 6x+9y=60 are equivalent) and are reduced to the form: ax + by = c whith integral coefficients and constant term such that GCF (a,b)=1.

2. Find an initial Solution: "Take advantage" of the fact that statements never contradict each other and thus system of equations constructed with both statements have always at least one solution. So resolve the system of equations.

3. Unicity: Once you arrive to a solution, say (x0, y0), go back to the first statement alone, for example, and check the unicity of the solution using only that statement by applying the test below. In case the solution is unique, statement 2 is superfluous and statement 1 is sufficient. The answer is A or D. In case the solution is not unique the answer is B, C or E.

Apply the test on statement (2). And update your answer.

If there is more than one solution using each statement alone then the answer is C.

-------------------------------------------------------------------------------------------------------------------------------------------------
Now here is the rule that indicates whether or not a non-negative integer solution is unique to an equation:
Suppose the equation be: ax+by=c (reduced with a, b, c positive integers. i.e. GCF(a,b)=1)
If (x0-b)<0 AND (y0-a)<0 then there is no other non-negative integer solution than (x0, y0) and the corresponding statement is sufficent.
If (x0-b)>=0 OR (y0-a)>=0 then other non-negative integers solutions exist and the statement is not sufficient.

If the variables must be positive the test is:
If (x0 - b)<=0 AND (y0 - a)<=0 then there is no other positive integer solution than (x0, y0) and the corresponding statement is sufficent.
If (x0 - b)>0 OR (y0 - a)>0 then other positive integers solutions exist and the statement is not sufficient.

Note: The test is to subtract each coefficient from the solution found for the opposite variable.
--------------------------------------------------------------------------------------------------------------------------------------------------
Let's apply this to a real GMAT problem:
A man buys some juice boxes. The boxes are from two different brands, A and B. How many boxes of brand A did the man buy if he bought $5.29 worth of boxes?
(1) The price of brand A box is $0.81 and the price of brand B box is $0.31
(2) The total amount of boxes is 9

Variables here must be positive integers -number of juice boxes- since it is suggested that some juice boxes are from brand A and the rest from brand B.
1. The equations provided are:
(1) 0.81A + 0.31B = 5.29.
(2) A + B = 9
Which are reduced to:
(1) 81A + 31B = 529
(2) A + B = 9,
which is a system of reduced, linear, non-equivalent equations.

2. Find an initial Solution:
(1) 81A + 31B = 529. GCF(31, 81)=1.
(2) A + B = 9

Mutliplying (2) by 31 and subtracting it from (1) we get:
50A=250 so A=5 and B=4.
An initial solution is (5, 4)

3. Unicity:
Unicity for statement (1):
81(5) + 31(4) = 529
Since
(5 - 31) <=0 AND (4 - 81)<=0 then there no other positive solution than (5, 4) so statement (1) is sufficient.

Unicity for statement (2):
It is obvious that statement (2) alone is not sufficient but the test is still applicable.
1(5) + 1(4) = 9
Since
(5 - 1) >0 OR (4 - 1) > 0 then there are other positive solutions than (5, 4) so statement (2) is not sufficient.
Answer A.

Hope this helps.


Would this still be the case if the equation were written 0.81x + 0.31y = 5.29
(x,y) remains (5,4)
but (5 - 0.31) AND (4-0.81) are both greater than 0
but there is no other solution
Intern
Intern
Joined: 13 Apr 2017
Posts: 42
Own Kudos [?]: 9 [0]
Given Kudos: 28
Send PM
Re: Joanna bought only $0.15 stamps and $0.29 stamps. How many $0.15 stamp [#permalink]
udaymathapati wrote:
Joanna bought only $0.15 stamps and $0.29 stamps. How many $0.15 stamps did she buy?


(1) She bought $4.40 worth of stamps.

(2) She bought an equal number of $0.15 stamps and $0.29 stamps.


Another solution to find statement 1 only has one valid pair of values:

Let C = number of $0.15 stamps purchased
Let E = number of $0.29 stamps purchased

Statement 1:
0.15C + 0.29E = 4.40 or 15C + 29E =440
-> C = (440-29E)/15
OR C = (450-30E+E-10)/15 = 30-2E +(E-10)/15;
C is integer, then E must be 10,25,... but E can not be 25 because 29*25 > 440
-> E must be only 10
-> Statement 1: Sufficient
GMAT Club Bot
Re: Joanna bought only $0.15 stamps and $0.29 stamps. How many $0.15 stamp [#permalink]
   1   2 
Moderator:
Math Expert
92881 posts

Powered by phpBB © phpBB Group | Emoji artwork provided by EmojiOne